#12 Rosh Review

अब Quizwiz के साथ अपने होमवर्क और परीक्षाओं को एस करें!

Question: Approximately what percentage of pregnant females with untreated asymptomatic bacteriuria will go on to develop pyelonephritis?

Answer: 30%.

Question: Why should beta-blockers be avoided in the treatment of patients with cocaine related chest pain or hypertension?

Answer: Administration of a beta-blocker can cause unopposed alpha effects leading to worsening symptoms and blood pressure. Rapid Review Asymptomatic Hypertension MC causes of BP elevation: pain, anxiety No workup indicated Outpatient follow-up

Question: What classic corneal finding is seen with herpes simplex keratitis?

Answer: Dendritic lesions. Rapid Review Ultraviolet Keratitis Patient will be a skier or welder Complaining of bilateral decreased visual acuity, pain and redness Fluorescein staining will show multiple punctate lesions Treatment is cycloplegics

Question: Children with hereditary spherocytosis are at risk for aplastic crisis as result of an infection from which virus?

Answer: Parvovirus B19.

Question: What bacterial pathogen is implicated in hordeolum and chalazion?

Answer: Staphylococcus species. Rapid Review Dacrocystitis Nasolacrimal duct obstruction Inframedial (dacroadenitis: supratemporal) Rapid onset Unilateral pain/redness Epiphoria

Question: How common is hearing loss in Ramsay Hunt syndrome?

Answer: Up to 50% of people experience ipsilateral hearing loss. Rapid Review Herpes Zoster Age, immunodeficiency Reactivation of latent VZV in dorsal root ganglion Grouped vesicles on an erythematous base, dermatomal Postherpetic neuralgia, Ramsay Hunt syndrome, zoster ophthalmicus Acyclovir

Question: What layer of skin edema defines urticaria?

Answer: Urticaria involves localized dermal edema produced by transvascular fluid extravasation. Rapid Review Urticaria Urticaria=wheals=hives IgE mediated reaction to allergen Transient blanching, edematous papules and plaques Mild/moderate: antihistamines +/- steroids Severe: treat as anaphylaxis

A 69-year-old man with atherosclerosis presents with calf pain during ambulation. He notices a blanching of the leg skin when the pain is at its worst. If he stops walking, the pain ceases within 5 minutes. He has a 120 pack-year history of tobacco use. Examination of the toes reveals prolonged capillary refill but normal sensation. A recent work-up revealed normal cardiac function. Which of the following abnormalities would most likely be found during examination of this patient's lower extremities? Atrophic, shiny skin Intrinsic foot muscle weakness Pitting edema Varicose veins

Correct Answer ( A ) Explanation: Occlusive peripheral arterial disease results from atherosclerotic changes of vessels outside of the heart. It is heralded by intermittent claudication, a physical activity induced extremity pain that ceases with a few minutes of rest. Other classic symptoms include the 5 P's: pulselessness, pallor, pain, paresthesias and paralysis. Limb-threatening ischemia, suggested by paresthesias and paralysis, necessitate immediate evaluation and treatment. Prolonged capillary refill is also a common finding. Chronic poor blood supply causes the typical atrophic, shiny, dry skin changes seen with peripheral arterial disease. Evaluation includes pulse examination, abdominopelvic auscultation, Doppler assessment and ankle-brachial index testing. Diagnosis can be confirmed with Duplex ultrasonography, CT-angiography or MRI-angiography. Treatment begins with lifestyle modifications, namely tobacco cessation and regular exercise. Medications include lipid lowering statins, aspirin and other antiplatelet drugs like clopidogrel and cilostazol. Distal muscle weakness (B) and atrophy occurs more with peripheral neuropathy and myopathy than with peripheral arterial disease. If distal sensation is intact, peripheral neuropathy is less likely. Peripheral edema (C) occurs more commonly with cardiac failure, venous disturbance, lymphatic disturbance or infection than with peripheral arterial disease. It would not be the most expected finding in someone with a recent normal cardiac function test. Venous varicosities (D) suggest venous, not arterial, insufficiency, and rarely presents with intermittent claudication symptoms.

Which one of the following statements is an important feature of somatization disorder that helps differentiate it from conversion disorder, malingering, or hypochondriasis? Abdominal pain that is recurring over many years after multiple negative workups Physical symptoms disproportionate to a demonstrable organic disease Sudden dramatic onset of a single symptom, typically simulating some non-painful neurologic disorder where there is no anatomical explanation Symptoms that are intentionally produced in the absence of apparent external incentives

Correct Answer ( A ) Explanation: There are four types of specific disorders that are collectively called somatoform disorders: somatization disorder, conversion disorder, pain disorder, and hypochondriasis. Somatization refers to a tendency to experience and communicate psychological distress as physical symptoms in the absence of identifiable pathology. Patients with somatization disorders seek medical attention because they are convinced that their symptoms reflect real physical disease. In contrast to malingering and factitious disorder, the symptoms are neither feigned nor under the voluntary control of the patient. Depression and anxiety disorder are often present in these patients. They often present with multiple symptoms rather than a few specific symptoms. In conversion disorder (C) there is a sudden and dramatic onset of a single symptom without associated pathophysiological or anatomical explanation. The typical symptoms are that of a non-painful neurological disorder such as a pseudoseizure, syncope, coma, paralysis of a single limb, tremors and sensory loss in a limb. Patients describe these symptoms with apparent lack of concern for their gross symptoms. The presence of physical symptoms that are disproportionate to demonstrable organic disease (B) is a characteristic of hypochondriasis. Symptoms that are intentionally produced in the absence of apparent external incentives (D) is consistent with factitious disorder. Note: Illness anxiety disorder was derived in part from the diagnosis of hypochondriasis, which was eliminated from DSM-5. In DSM-5, patients previously diagnosed with hypochondriasis are nearly always diagnosed with either somatic symptom disorder (if physical complaints are prominent) or illness anxiety disorder (if physical complaints are minimal or nonexistent). Although relatively few studies of illness anxiety disorder have been published, there is a larger literature on the symptom of health anxiety, defined as persistent unrealistic worry or conviction about having an illness. The spectrum of health anxiety ranges from none to severe, with the severe end representing a clinical endpoint (such as illness anxiety disorder or hypochondriasis).

A 44-year-old woman presents to the emergency department complaining of abdominal pain, nausea, and vomiting. The pain began last night and woke her up from sleep approximately six hours ago. The pain is located in the epigastric area and the right upper quadrant of the abdomen and she describes it as a squeezing pain. Physical exam reveals a positive Murphy sign. What is the most likely diagnosis? Cholangitis Cholecystitis Duodenal ulcer Pancreatitis

Correct Answer ( B ) Explanation: Acute cholecystitis typically causes symptoms of pain in the epigastrium or right upper quadrant of the abdomen. Occasionally the pain radiates to the right scapula or right shoulder. Onset is typically sudden and the pain is constant and frequently associated with nausea with or without vomiting. Patients frequently complain of pain that awakens them from sleep. An ultrasound will show the presence of a thickened gallbladder wall and pericholecystic fluid. Treatment includes analgesic therapy and early elective laparoscopic cholecystectomy ideally within 24 hours of being admitted to the hospital. Broad-spectrum antibiotic administration is also recommended to prevent secondary bacterial infections. Pancreatitis (D) is typically associated with epigastric pain that radiates to the back. Cholangitis (B) presents with abdominal pain; however is typically associated with a change in mental status and other signs and symptoms associated with sepsis (e.g. fever). A duodenal ulcer (A) typically presents with pain in the epigastric area that is associated with hunger and relieved with food intake.

During the menstruation section of a gynecologic history, you determine that your 36-year-old patient's menstrual periods are increasing in length from four days to eight days, even though she still cycles every 28 days. She also reports more blood flow than typical during the first three days of these new eight day periods. Which of the following terms correctly defines this abnormal uterine bleeding? Amenorrhea Menorrhagia Metrorrhagia Polymenorrhea

Correct Answer ( B ) Explanation: Normal menstruation (less than 80 mL in total blood flow) lasts from two to seven days, and cycles between 24 and 35 days. Menorrhagia is defined as excessive bleeding (> 80 mL per cycle) or menstruation > 7 days, which is mainly due to an anatomic or hemostatic abnormality. There are many terms that describe menstrual variations according to duration, frequency, and amount of uterine bleeding: Amenorrhea (A) is the absence of bleeding for > 3 cycles. Metrorrhagia (C) is light uterine bleeding at irregular intervals. Menometrorrhagia is heavy uterine bleeding at irregular intervals. Oligomenorrhea is regular bleeding which cycles at intervals > 35 days. Polymenorrhea (D) is regular bleeding which cycles at intervals < 21 days. Dysfunctional uterine bleeding is a diagnosis of exclusion; refers to excessive, noncyclic bleeding which is mainly due to anovulation. Menorrhagia is excessive bleeding (> 80 mL per cycle) or menstruation > 7 days, which is mainly due to an anatomic or hemostatic abnormality.

A mother who had no prenatal care brings her 3-day-old infant to the ED secondary to eye drainage. On exam, the child has marked swelling and redness of the eyelids, severe chemosis, and purulent discharge bilaterally. Which of the following organisms is the most likely diagnosis? Chlamydia trachomatis conjunctivitis Neisseria gonorrhea conjunctivitis Staph. aureus conjunctivitis Vernal conjunctivitis

Correct Answer ( B ) Explanation: This neonate has N. gonorrhea conjunctivitis. This is an extremely aggressive form of conjunctivitis that can ulcerate and perforate an intact cornea within hours or days. It is considered an ocular emergency and is most commonly seen in newborns (ophthalmia neonatorum), typically within the first 3-5 days of life. It is also seen occasionally in sexually active adults. The discharge is characteristically purulent. Treatment includes parenteral and topical antibiotics. A Gram stain and culture should also be obtained. C. trachomatis (A) conjunctivitis is the leading cause of preventable blindness in the world. It is also commonly seen in newborns around 1-2 weeks after birth. Gram stain will be negative because Chlamydia trachomatis is an intracellular parasite. In newborns, it may also be associated with chlamydial pneumonia. Staph. aureus (C) conjunctivitis is also associated with mucopurulent discharge, but it is not typically seen in the neonatal period. White ulcers may be seen at the limbus (marginal ulcers) as a result of an allergic reaction to the staphylococcal toxin. Vernal conjunctivitis (D) is a type of allergic conjunctivitis that most commonly affects male children. It is characterized by intense itching, burning photophobia, chemosis, bilateral lid edema, cobblestone papillae under the upper lid, and a stringy mucoid discharge. A potential complication is a "shield ulcer" produced from the irritated papillae and discharge; if the ulcer becomes infected or scarred, loss of vision may occur.

A previously healthy 25-year-old woman presents to your office with complaints of flank pain and burning with urination. She tells you that two days ago she started experiencing symptoms that also included frequent urination, nausea, vomiting and chills. Her temperature is 102°F and she has costovertebral angle tenderness on physical exam. Which of the following is the most appropriate therapy? Amoxicillin-clavulanate Ciprofloxacin Fosfomycin Nitrofurantoin

Correct Answer ( B ) Explanation: Urinary tract infections (UTIs) are differentiated into two types. Cystitis is an infection of the bladder and pyelonephritis is an infection of the kidneys. Pyelonephritis is a more serious infection than cystitis and the antibiotics to treat the infection are different than those used with cystitis. Patients present with symptoms similar to cystitis such as dysuria, urgency, and urinary frequency, but also have fever, chills, nausea, vomiting, flank pain and costovertebral angle tenderness. Mild to moderate uncomplicated pyelonephritis may be managed on an outpatient basis. Fluoroquinolones such as ciprofloxacin or levofloxacin are commonly used to treat outpatient pyelonephritis. Another option is trimethoprim-sulfamethoxazole. Patients considered to have complicated pyelonephritis including pregnant women, diabetics, immunocompromised, and those with renal failure or hospital acquired infection should be treated in the inpatient setting with parenteral antibiotics. Amoxicillin-clavulanate (A) is a beta lactam that may be used in the treatment of cystitis. Oral beta lactam agents are less effective than other agents for treatment of pyelonephritis. If the pathogen is susceptible and an oral beta lactam agent is continued, it should be administered for at least 14 days. Fosfomycin (C) and nitrofurantoin (D) are also used in treatment of cystitis, but should be avoided in pyelonephritis as they do not achieve sufficient renal tissue levels.

A two-year-old boy develops pallor following a viral upper respiratory tract infection. His serum studies reveal a normocytic anemia and the presence of spherocytes on the peripheral blood smear. Which of the following tests can be done to confirm the diagnosis? Direct Coombs test Hemoglobin electrophoresis Osmotic fragility test Serum reticulocyte count

Correct Answer ( C ) Explanation: Hereditary spherocytosis is a clinical diagnosis based on family history, peripheral blood smear, and presence of splenomegaly. Blood film should show spherocytes and reticulocytes due to the high turnover of red blood cells from hemolysis. The presence of spherocytes in the blood can be confirmed with an osmotic fragility test. This is done by incubating red blood cells in hypotonic saline in increasing dilutions. This causes the red blood cells to swell and eventually burst. The spherocytes more readily lyse than normal cells in hypotonic solutions. Clinical correlation is necessary since this test is not specific, and normal results can be present in up to 20 percent of patients. Direct Coombs test (A) is done to detect cell lysis resulting from an autoimmune process. The test detects the presence of antibodies directed at a patient's own red blood cells. Hemoglobin electrophoresis (B) is used to detect different types of hemoglobin and is used in the diagnosis of thalassemias and sickle cell disease. Serum reticulocytes (D) are elevated in many conditions that result in high red blood cell turnover. The presence of reticulocytosis alone does not confirm or exclude a diagnosis.

A 5-year-old boy has acute onset of hematuria, periorbital edema, and hypertension. He has no other complaints and review of systems is unremarkable. Recent medical history is significant for a "cold" last week. What is the most likely etiology of his hematuria? Coagulopathy IgA nephropathy Post-streptococcal glomerulonephritis Urinary tract infection

Correct Answer ( C ) Explanation: Post-streptococcal glomerulonephritis classically presents as hematuria that begins 1-2 weeks after an episode of streptococcal pharyngitis or 3-6 weeks after a streptococcal skin infection. Often the initial infection is mild and thus does not lead to medical evaluation or recognition. Affected children are typically 5 - 12 years old with a mean age of 7 years old. Treatment is supportive. Although > 60% of patients briefly develop hypertension, > 95% make a full recovery. Inherited or acquired coagulopathy (A) may also present with acute hematuria. However, other symptoms of coagulopathy almost always occur prior to the onset of microscopic or gross hematuria. IgA nephropathy (B) may also cause gross hematuria. However, hematuria caused by IgA nephropathy begins within 1-2 days of the onset of upper respiratory or gastrointestinal infection. Urinary tract infection (D) may cause hematuria, but symptoms of dysuria, urinary frequency, and urinary urgency are more common. Moreover, urinary tract infections are rare in school-aged boys.

A 21-year-old woman presents with sudden onset of right-sided hearing loss for 1 day. She also has severe vertigo associated with nausea and vomiting. The patient reports URI symptoms over the previous 3 days. Currently, she feels fullness and mild otalgia in her right ear and abnormal taste when she eats food. On exam, you note a few vesicles anterior to her right ear. She has drooping of the right side of her face and cannot wrinkle the right side of her forehead. Which of the following is the best treatment for this patient? Antibacterial otic drops Head CT scan and neurology consult Herpes zoster vaccine Prednisone and acyclovir

Correct Answer ( D ) Explanation: Ramsay Hunt syndrome is an acute peripheral facial neuropathy associated with an erythematous vesicular rash of the skin of the ear canal, auricle (also termed herpes zoster oticus), or mucous membrane of the oropharynx. It is caused by reactivation of herpes zoster in the geniculate ganglion. This nerve cell ganglion is responsible for movements of the facial muscles, sensation of parts of the ear and ear canal, taste function of the tongue, and lubrication of the eyes and oral mucosa. Because the vestibulocochlear nerve is close to the geniculate ganglion, patients may complain of tinnitus, hearing loss, and vertigo. Patients usually present with paroxysmal pain deep within the ear. The pain often radiates outward into the pinna of the ear and may be associated with a more constant, diffuse, and dull background pain. The onset of pain usually precedes the rash by several hours and even days. Poor prognostic factors for good functional recovery include age older than 50 years, complete facial paralysis, and lack of CN VII nerve excitability. Combined therapy with prednisone and acyclovir are the most commonly used and are most effective if started within the first 3 days of symptom onset. Antibacterial otic drops (A) are ineffective for a viral infection. In some cases, the ulcers from herpes zoster lead to a secondary bacterial infection. In these cases, antibiotics otic drops may be necessary. A CT of the head (B) is an appropriate workup when there is a clinical suspicion for a central cause of facial droop (occurs with forehead sparing). However, Ramsay Hunt syndrome involves a peripheral neuropathy, and CT imaging is not necessary. Herpes zoster vaccine (C) is used to help the reaction of the Varicella-Zoster virus. The vaccine is recommended for use in people 60 years old and older to prevent zoster and has been shown to reduce the risk of shingles by approximately 50% and the risk of postherpetic neuralgia by 67%. It does not have a role in the treatment of Ramsay Hunt syndrome.

Question: What is the calculation for absolute neutrophil count (ANC)?

Answer: ANC = (total WBC) x (percentage of neutrophils and bands). Rapid Review Neutropenic Fever > 38.3ºC + ANC < 500 Obtain cultures Rx: admission, ABX

Question: What is the most common complication of post-streptococcal glomerulonephritis?

Answer: Hypertension.

Question: What type of metal is responsible for forming rust rings?

Answer: Iron.

Question: What is the preferred treatment of individuals with incidentally found asymptomatic gallstones found?

Answer: No treatment as the probability of developing symptoms within five years is low. Rapid Review Cholecystitis Patient will be complaining of colicky, steadily increasing RUQ or epigastric pain after eating fatty foods PE will show Murphy's sign, Boas sign Diagnosis is made by: Initial: US, Gold standard: HIDA Most commonly caused by obstruction by a gallstone Treatment is cholecystectomy

A 36-year-old woman is undergoing chemotherapy for her breast cancer. She is very concerned about the risk of neutropenic fever and wonders if the recent appearance of myalgias and a low-grade temperature (38°C) are signs of this. How many days after her chemotherapy session should she experience a nadir in her absolute neutrophil count? 12 days 15 days 3 days 7 days

Correct Answer ( D ) Explanation: For the majority of chemotherapy regimens, the nadir in the absolute neutrophil count (ANC) will occur five to ten days after completion of a chemotherapy session. The nadir of the absolute neutrophil count is less likely to occur at 3 (C), 12 (A), or 15 (B) days.

Question: Which deficiency of the non-specific limb of the immune system has been clearly associated with meningococcal sepsis and meningitis?

Answer: Complement deficiency C5-C9, properdin, factor H, or factor D has been associated with a special susceptibility to infections caused by N. meningitides.

Question: What virus is most commonly associated with Burkitt lymphoma?

Answer: Epstein-Barr virus. Rapid Review Lymphoma NHL Multiple, peripheral nodes; noncontiguous spread Extranodal involvement Associated with EBV Hodgkin's Localized, single group of nodes; contiguous spread Age distribution: 15-35 and > 55 B symptoms Reed-Sternberg cells

Question: Vaginal delivery is contraindicated with which active sexually transmitted disease?

Answer: Herpes vaginalis infection. Rapid Review Neisseria Gonorrhea Conjunctivitis Newborns < 5 days old, sexually active adults Hyperpurulent discharge Corneal ulceration/perforation Admission Topical + IV antibiotics

Question: Large pericardial effusions (>250 ml) have what classic appearance on chest radiograph?

Answer: Large "water-bottle" cardiac silhouette with epicardial halo. Rapid Review Light's Criteria Fluid is exudate if one of the following Light's criteria is present: Fluid protein/serum protein ≥ 0.5 Fluid lactate dehydrogenase (LDH)/serum LDH ≥ 0.6 Fluid LDH level > 2/3 the upper limit of normal of serum LDH

Question: Peripheral arterial disease most commonly affects which vessels?

Answer: The superficial femoral artery (calf pain) and aortoiliac system (thigh/buttock pain).

A 24-year-old man with HIV presents with severe dysphagia and odynophagia for 1 week. Physical examination reveals the above image. What management is indicated? A Acyclovir B Clotrimazole troches C CT scan of the neck D Fluconazole

Correct Answer ( D ) Explanation: This patient presents with infectious esophagitis secondary to a candidal infection and can be treated with fluconazole. Esophagitis is defined as inflammation of the esophagus and can be caused by a number of mechanisms including infection, allergy related (eosinophillic) or pills. Infectious esophagitis typically occurs in patients that are immunocompromised (diabetes, HIV/AIDS, chronic steroids etc.). Esophageal candidiasis is common in the HIV/AIDS population and is considered an AIDS-defining illness. In addition to candida, it can be caused by HSV1 and CMV. Patients with infectious esophagitis present with severe odynophagia that may be to solids, liquids or both. They may also have chest pain, nausea and dyspepsia. Dehydration is common secondary to pain. Diagnosis can be made based on clinical presentation but an endoscopy can be performed to confirm the diagnosis. In immunocompromised patients with candidal esophagitis, treatment should be with fluconazole for 14 to 21 days. If they are unable to tolerate oral medications, intravenous fluconazole can be given. Acylovir (A) is used in the treatment of herpes esophagitis. Clotrimazole troches (B) can be used in the treatment of mild oropharyngeal candidiasis in immunocompetent patients. CT scan of the neck (C) is unnecessary and is unlikely to show the pathology in the esophagus.

A 55-year-old mechanic complains of irritation in his right eye lasting for 2 days. On direct visualization you see a small, dark foreign body on the periphery of the cornea and are able to remove it without complications. However, there is a patch of reddish-brown discoloration extending several millimeters around the area where the foreign body had been. Which one of the following is most appropriate for this patient? An antibiotic ointment to be used every 2-4 hours Irrigation with 0.9% saline solution under pressure Prompt ophthalmologic evaluation Watchful waiting

Correct Answer ( C ) Explanation: Corneal foreign bodies generally fall under the category of minor ocular trauma. Small particles may become lodged in the corneal epithelium or stroma, particularly when projected toward the eye with considerable force. The foreign object may set off an inflammatory cascade, resulting in dilation of the surrounding vessels and subsequent edema of the lids, conjunctiva, and cornea. White blood cells also may be liberated, resulting in an anterior chamber reaction or corneal infiltration. If not removed, a foreign body can cause infection or tissue necrosis. If a metal foreign body is present on the cornea for more than 24 hours a rust ring will often be present in the superficial layer of the cornea. This material is toxic to the cornea and should be removed as soon as possible. Rust rings that remain in the cornea after removal of a metallic foreign body may require removal with a slit lamp and rust ring drill. Referral to an ophthalmologist within 24-48 hours is the best management in this case. Antibiotics (A) may be prescribed to prevent infection of an open corneal abrasion, however removal of the rust ring is the crucial step to treatment. Irrigation with 0.9% saline solution (B) under pressure and watchful waiting (D) are not sufficient when a toxic material such as metal in introduced to the eye.

A six-year-old boy presents to his pediatrician's office with a 12-hour history of headache, stiff neck, fever to 104°F, and lethargy. A complete blood count and differential, urinalysis, blood culture, and electrolytes are ordered. While under observation, awaiting the results of the blood tests, in the doctor's office, the boy begins vomiting and develops a petechial rash from head to feet followed rapidly by the formation of necrotizing purpura. Two hours later, while awaiting transport to a children's hospital, he develops shock, has a cardiac arrest, and dies. What is the most likely diagnosis, from the list below? Idiopathic thrombocytopenia Leukemia Meningococcal meningitis Viral encephalitis

Correct Answer ( C ) Explanation: Meningococcal meningitis caused by Neisseria meningitides is one of the most feared and rapidly fulminant infectious diseases. Although meningococcal sepsis can follow an even more rapid course than meningococcal meningitis, the headache and stiff neck are consistent with meningeal involvement. The rapid course in this patient, with a marked fever rapid development of the petechial rash with progression to necrotizing purpura are classic for meningococcal sepsis, with or without meningitis. The skin lesions are a manifestation of cutaneous vasculitis from which the organism can be demonstrated microscopically or on culture. The shock, cardiac arrest, and death are a manifestation of endotoxemia associated with the bacterial cell wall of these Gram-negative organisms. Idiopathic thrombocytopenia (A) is manifest by a low platelet count, low enough to allow bleeding into the skin through small capillaries. This disease is rarely associated with a high fever, necrotizing purpura, and a fulminant course leading to death in a matter of hours. Leukemia (B) can rarely present with overwhelming sepsis, but it usually has a history of weeks of associated symptoms such as pallor, anorexia, low-grade fever, malaise, etc. Viral encephalitis (D) also has a less fulminant course with days of headache, waxing and waning neurological signs and symptoms, and although petechia may occur as a manifestation of cutaneous viral vasculitis, it is slow to progress all over the body and almost never associated with necrotizing purpura. Although cardiac arrest may occur with a seizure, endotoxic shock, and death are not a part of viral encephalitis.

Question: What percentage of children with untreated Kawasaki Disease develops coronary artery aneurysms?

Answer: 15-25%. Rapid Review Kawasaki Disease Patient will be a child < 4 years old With a history of high fever for 5 days Complaining of conjunctivitis, rash, adenopathy, strawberry tongue, hand/feet edema, fever Treatment is IVIG + aspirin Comments: #1 cause of pediatric acquired heart disease, risk for coronary artery aneurysm Mnemonic: CRASH and burn: Conjunctivitis, Rash, Adenopathy, Strawberry tongue, Hand/feet edema, Fever

Question: When do you start treatment for stage 3 and 4 CLL?

Answer: At time of diagnosis. Rapid Review Chronic Lymphocytic Leukemia (CLL) Patient will be > 60 years old PE will show generalized lymphadenopathy Labs will show isolated lymphocytosis, smudge cells Comments: Most common adult leukemia in western world and generalized lymphadenopathy > 60 years old

Question: What are some treatments for essential tremor?

Answer: Beta-blockers or anti-convulsants may reduce benign essential tremor. Rapid Review Essential Tremor Patient with a history of a family member with similar symptoms Complaining of hand tremor that is exacerbated by action and improved after alcohol consumption Most commonly caused by autosomal dominant Treatment is propanolol

Question: What is the correlation between etiology and bilateral conjunctivitis?

Answer: Bilateral symptoms are usually due to infections or allergies, while unilateral symptoms are usually due to chemicals, trauma or toxins. Rapid Review Bacterial Conjunctivitis Patient will be complaining of red/pink eye with discharge usually worse in the morning PE will show purulent (yellow) discharge and crusting Most commonly caused by Staph aureus, Strep pneumoniae, H.Flu. Contact lens wearers: Pseudomonas aeruginosa Treatment is topical antibiotic drops

Question: Which alternative therapy has shown benefit in patients with anxiety and tinnitus?

Answer: Biofeedback. Rapid Review Tinnitus Acoustic neuroma: CN VIII, hearing loss + tinnitus + disequilibrium Meniere's disease: recurrent vertigo + tinnitus + hearing loss ​Ramsay Hunt syndrome: facial paralysis, zoster lesions, tinnitus Labyrinthitis: sudden severe vertigo, hearing loss, tinnitus, not recurrent Head trauma Electrical injury Diving Ototoxic agents: Salicylates: respiratory alkalosis + anion gap metabolic acidosis + tinnitus NSAIDs Quinine ABX (aminoglycosides, erythromycin, vancomycin) Chemotherapeutic agents

Question: What is the classic diagnostic finding for achalasia?

Answer: Bird-beak esophagus seen on barium swallow that represents dilation of the upper esophagus with narrowing of the lower esophagus. Rapid Review Achalasia Patient will be complaining of dysphagia to solids and liquids PE will show absent peristalsis in lower esophagus Barium swallow shows "bird beak" appearance Diagnosis is made by esophageal manometry increased LES pressure

Question: Which antidepressant is contraindicated in patients with eating disorders?

Answer: Bupropion. Rapid Review Bulimia Recurrent binge eating Overconcern with body shape/weight Weight: normal or above normal Submandibular gland enlargement Calluses over dorsal aspect of fingers Dental enamel erosion Rx: cognitive behavioral therapy (first-line), SSRIs (second-line)

Question: What are the most common fungal causes of purulent pericarditis?

Answer: Candida and histoplasma species. Rapid Review Histoplasmosis Patient with a history of travel to Ohio/Mississippi river valleys and exposure to bird/bat droppings X-ray will show solitary pulmonary calcification, hilar and mediastinal adenopathy Diagnosis is made by culture Treatment is itraconazole or amphotericin B

Question: What is the leading cause of mortality in patients with diabetes?

Answer: Cardiovascular disease. Rapid Review Diabetic Ketoacidosis Patient will be a diabetic With a history of Infection, Ischemia (cardiac, mesenteric), Infarction, Ignorance (poor control), Intoxication (FIVE I's) Complaining of abdominal pain, vomiting, and fatigue PE will show hyperglycemia and ketonemia Labs will show anion gap metabolic acidosis Treatment is IV fluids & insulin infusion Comments: Corrected sodium add 1.6 mEq/L for each 100mg/dL in serum glucose

Question: What does postcoital (post sexual intercourse) bleeding suggest?

Answer: Cervical cancer, which must be ruled out. Rapid Review Dysfunctional Uterine Bleeding (DUB) MCC of abnormal vaginal bleeding in reproductive women Menarche, perimenopause Anovulatory: ↑ Estrogen, ↓ progesterone → endometrial hyperplasia/bleeding Unpredictable bleeding Ovulatory: Predictable bleeding Dx of exclusion Rx: combination OCPs Unstable bleeding: IV estrogen

Question: What is the other name for vitamin B12?

Answer: Cyanocobalamin. Rapid Review Vitamin B12 (Cobalamin) Deficiency Patient will be a vegan Complaining of fatigue, weakness, and peripheral neuropathy PE will show pallor and glossitis Labs will show MCV > 100, hypersegmented neutrophils, elevated homocysteine, elevated methylmalonic acid Treatment is parenteral vitamin B12 Comments: only vitamin B12 deficiency results in neurological symptoms

Question: What is the initial treatment for prolonged bleeding in patients with von Willebrand disease?

Answer: Desmopressin (DDAVP), which causes release of VWF from cells. Rapid Review von Willebrand Disease Most common inherited bleeding disorder Decreased protein (WWF) causes abnormal platelet adhesion Increased bleeding time Treatment with ​desmopressin (DDAVP)

Question: Why must alcohol be avoided during metronidazole therapy?

Answer: Disulfuram-like effect. Rapid Review Trichomoniasis Patient will be a woman complaining of malodorous vaginal discharge PE will show frothy, green/yellow discharge, "Strawberry cervix" Labs will show pH >5, flagellated, motile, pear shaped Diagnosis is made by wet mount Most commonly caused by Trichomonas vaginalis Treatment is metronidazole

Question: What is the name of the maneuver used to diagnose benign paroxysmal peripheral vertigo?

Answer: Dix-Hallpike Maneuver.

Question: Which is the more common cause of interstitial nephritis—drugs or infectious diseases?

Answer: Drugs. They cause over 70% of cases.

Question: What study is diagnostic for pill esophagitis?

Answer: Esophagoscopy. Unfortunately, pill esophagitis will not be apparent on any imaging modality. Rapid Review Esophagitis Most common cause: GERD Infectious esophagitis: immunocompromise Infectious etiologies: Candida > HSV, CMV Odynophagia, dysphagia Oral lesions not always present Treatment: Candida: fluconazole CMV: Ganciclovir HSV: acyclovir

Question: What is the most common cause of neonatal hemorrhage?

Answer: Failure to administer vitamin K in the immediate postpartum period (associated with home births). Rapid Review Swallowed Maternal Blood Cracked nipple Apt test: differentiates fetal from maternal blood

Question: What are the risk factors for ectopic pregnancy?

Answer: History of pelvic inflammatory disease, tubal ligation, prior ectopic pregnancy, intrauterine device, assisted reproductive techniques, and in vitro fertilization. Rapid Review Ovarian Torsion Patient will be a woman age 15-30 or postmenopausal Complaining of sudden onset of unilateral (right > left) abdominal and pelvic pain Labs will show leukocytosis Diagnosis is made by TVUS. Gold standard: Laparoscopy Most commonly caused by cyst or tumor Treatment is emergent surgery

Question: What is the treatment of choice in those who develop Erythema multiforme?

Answer: In most cases of EM, supportive treatment is all that is necessary. Oral antihistamines, steroids, and analgesics may be beneficial. Rapid Review Erythema Multiforme (EM) Patient will be complaining of acute onset of a symmetric target lesions on palms and soles, face and trunk may also be involved PE will show target-like with a central dark papule surrounded by a pale area and a "halo" of erythema Most commonly caused by herpes simplex virus (HSV) Treatment is usually self-limiting, supportive Comments: Common drugs that cause EM: Sulfa, Oral hypoglycemics, Anticonvulsants, Penicillin, NSAIDs (SOAPS)

Question: Which side are varicoceles more common?

Answer: Left side. Rapid Review Phimosis Inability to retract foreskin behind glans Rx: hygiene, topical steroids Rx if vascular compromise: dorsal slit Definitive rx: circumcision

Question: Which class of medications should be given initially for acute pulmonary edema?

Answer: Nitrates. Rapid Review Acute Decompensated Heart Failure Exertional dyspnea, orthopnea, paroxysmal nocturnal dyspnea, pitting edema S3 ↑ BNP CXR: cardiomegaly, cephalization, Kerley B lines, effusions Most useful study: echo Treatment: BiPAP: ↑ oxygenation, ↓ work of breathing, ↓ preload/afterload Nitroglycerin: ↓ preload/afterload Furosemide: diuresis Hypotension without signs of shock: dobutamine (may worsen hypotension) Severe hypotension with signs of shock: norepinephrine (↑ systemic vascular resistance, ↑ HR, ↑ BP, ↑ myocardial O2 demand)

Question: What common conditions are associated with carpal tunnel syndrome?

Answer: Obesity, diabetes, hypothyroidism, pregnancy, and renal failure. Rapid Review Peripheral Neuropathies Sciatic: buttock injury, hip dislocation, ↓ knee flexion, foot drop, rx: ankle splint Common peroneal: proximal fibula injury, footdrop, rx: ankle splint Radial: crutches, wrist/finger drop, rx: wrist splint Ulnar: elbow injury, ↓ finger adduction/thumb grasp, 4th/5th digit paresthesias Lateral femoral cutaneous: inguinal ligament entrapment, upper thigh dysesthesia/numbness

Question: What is the most common goal of malingering patients in the emergency department?

Answer: Obtaining drugs. Rapid Review Somatoform Disorders Genuine sx with no identifiable cause F > M Conversion disorder: sudden loss of sensory or motor function Hypochondriasis: Preoccupation with having illness Pain disorder Rx: reassurance, minimize providers

Question: Which nutritional deficiency also presents with a malar rash?

Answer: Pellagra (vitamin B3 deficiency). Rapid Review Systemic Lupus Erythematosus (SLE) African-Americans, females Malar rash Arthritis Renal disease Cardiac: fibrinous pericarditis, Libman-Sacks endocarditis CNS: HA, stroke, seizures Drug induced: Hydralazine, INH, Procainamide, Phenytoin, Sulfonamides (HIPPS) Antinuclear antibodies (ANA): 100% sensitive, not specific Anti-dsDNA antibodies: 100% specific, poor prognosis Anti-Smith antibodies: 100% specific, not prognostic Antihistone antibodies: sensitive for drug-induced lupus NSAIDs, steroids, immunosuppressants, hydroxychloroquine

Question: What is the most common cause of dysentery in the United States?

Answer: Shigella sonnei. Although there are four Shigella species, S. sonnei is the most common and has only one known subtype. Other species include S. dysenteriae, S. flexneri, and S. boydii. Each has multiple subtypes. Rapid Review Shigellosis Patient will be complaining of fever, bloody, mucoid diarrhea and seizures (more common in children) Labs will show fecal RBCs and WBCs Treatment is ciprofloxacin Complications: HUS, Reiters syndrome

Question: What other bacterial agents are commonly associated with reactive arthritis?

Answer: Shigella, Salmonella, Yersinia and Campylobacter organisms. Rapid Review Reiter Syndrome (Reactive Arthritis) Patient with a history of recent GI or Chlamydia infection Complaining of acute asymmetric arthritis PE will show conjunctivitis, arthritis, urethritis Labs will show HLA-B27 Treatment is NSAIDs Comments: "Can't see" (uveitis), "can't pee" (urethritis), "can't climb a tree" (arthritis)

Question: Which rheumatological condition places patients at an increased risk of developing sialadenitis?

Answer: Sjogren's syndrome increased this risk due to a chronic setting of xerostomia. Rapid Review Sialadenitis Patient will be complaining of mouth swelling and pain that is worse with meals Most commonly caused by Staphylococcus aureus Treatment is antibiotics, warm compresses, gland massage, sialogogues

Question: What is the classic finding in pyloric stenosis seen on upper GI series?

Answer: The "string sign" which represents the slow passage of contrast through a narrowed pyloric sphincter. Rapid Review Pyloric Stenosis Patient will be 2 - 6 weeks old Complaining of non-bilious projectile vomiting after feeding and early satiety PE will show RUQ olive-like mass (hypertrophied pylorus) Labs will show hypochloremic hypokalemic metabolic alkalosis Diagnosis is made by ultrasound or UGI series (string sign) Treatment is surgical

Question: Though all infants are recommended to receive pertussis vaccination with a DTaP, what form of vaccination is recommended for adults?

Answer: The CDC recommend that all adults, including those over 64 years of age, receive a booster with a Tdap once during their lifetime. Rapid Review Pertussis ("Whooping Cough") Patient with a history of nasal congestion, cough, and low-grade fever Complaining of "rapid fire," repetitive coughing followed by an inspiratory "whoop" and post-tussive emesis Most commonly caused by Bordetella pertussis Treatment is macrolide - azithromycin

Question: What is the definition of Sepsis Criteria?

Answer: The presence of SIRS + a suspected or confirmed source of infection. Rapid Review Sepsis Syndromes SIRS: 2 or more of the following T >38° or <36° HR > 90 bpm RR > 20 bpm or PCO2 < 32 mmHg WBC >12000/mm3 or <4000/mm3 Sepsis: SIRS + infection Severe sepsis: sepsis + organ dysfunction Septic shock: sepsis + refractory hypotension

Question: Atrial-based pacemakers are recommended in all patients with symptomatic bradycardia except for which subclass?

Answer: Those who also have chronic atrial fibrillation. Rapid Review Sick Sinus Syndrome (SSS) SA node dysfunction Tachycardia-bradycardia syndrome: sinus rate varies from fast to slow and back again Syncope, palpitations Definitive rx: pacemaker placement + rate control medication Untreated SSS → sinus block or sinus arrest

Question: What is the most valuable laboratory test for diagnosing urinary tract infection?

Answer: Urinalysis. Rapid Review Acute Pyelonephritis Patient will be complaining of fever, dysuria, and flank pain PE will show CVA tenderness Urinalysis will show leukocyte esterase, nitrites and microscopy will show WBCs. Most commonly caused by Escherichia coli Treatment is ciprofloxacin Inpatient/pregnant: ampicillin/gentamicin or 3rd gen cephalosporin

Question: What is the most valuable laboratory test for diagnosing urinary tract infection?

Answer: Urinalysis. Rapid Review Acute Pyelonephritis Patient will be complaining of fever, dysuria, and flank pain PE will show CVA tenderness Urinalysis will show leukocyte esterase, nitrites and microscopy will show WBCs. Most commonly caused by Escherichia coli Treatment is ciprofloxacin Inpatient/pregnant: ampicillin/gentamycin or 3rd gen cephalosporin

Question: How soon after blood transfusion initiation will anaphylaxis occur?

Answer: Within minutes. It is triggered by presence of IgA antibodies in IgA-deficient patients. Rapid Review Transfusion Complications Massive transfusion: coagulopathy, hypothermia, hypocalcemia Febrile reaction: most common complication, fever/chills, Hemolytic reaction: ABO incompatibility, immediate fever/chills, HA Rx: stop transfusion, IVF, diuretics Allergic reaction: urticaria or hives TRALI: like ARDS Rx: stop transfusion Delayed reaction: 3-4 weeks after transfusion, decreased Hb, increased Br GVHD: immunocompromise, rash, pancytopenia, increased LFTs Prevention: irradiated blood products in immunocompromised

A 30-year-old woman presents to the Emergency Department with left lower quadrant pain. The pain started suddenly while she was sleeping and is rated a 10/10 in severity. Which of the following tests will yield a definitive diagnosis of an ovarian torsion? Computed tomography Magnetic resonance imaging Surgical visualization Ultrasound

Correct Answer ( C ) Explanation: Ovarian torsion is a twisting of the ovary around the vascular pedicle which results in ischemia. Most commonly there is an enlarged cyst or mass which predisposes the ovary to torsion. Risk factors include ovarian cysts or masses ≥ 4 cm, prior pelvic surgery, history of tubal ligation, fertility treatments, and pregnancy. Due to its enlarged tubo-ovarian ligament length, the right ovary is the most common side of torsion. Presenting history will include sudden onset and severe, unilateral pelvic pain, commonly during strenuous activity, as well as nausea and vomiting. Definitive diagnosis is surgical. Doppler ultrasound showing absence of flow within the ovary is highly specific, however, the presence of blood flow does not rule out ovarian torsion, as the ovary has a dual blood supply and may maintain blood flow even in the presence of torsion. The most common finding in ovarian torsion is an enlarged, edematous ovary. Treatment is immediate and emergent surgical intervention in order to salvage the ovary and prevent secondary adverse events such as hemorrhage, peritonitis, and adhesions. Neither computed tomography (A) (CT) or magnetic resonance imaging (B) (MRI) are recommended as primary modalities to evaluate a patient with suspected ovarian torsion, however, both are useful in making a diagnosis more likely by visualization of an enlarged ovary with an ovarian mass or cyst, an edematous ovary in an abnormal location, lack of enhancement of the ovary after administration of intravenous contrast, or coiled ovarian vessels. Ultrasound (D), both pelvic and transabdominal, is the method of choice to evaluate a woman in whom an ovarian torsion is suspected. Large masses and cysts are quickly and easily identified on ultrasound. The lack of blood flow, as witnessed by lack of doppler flow into an ovary, is highly specific for ovarian torsion, as is visualization of the "whirlpool sign" created by the torsed pedicle of the ovary. The appearance of blood flow does not rule out an ovarian torsion as the ovary has a dual blood supply and may still provide some perfusion despite torsion. Despite all these findings, the definitive diagnosis is considered to be surgical visualization.

Which of the following is classified as a dysmotility disorder? Achalasia Esophageal web Schatzki ring Zenker diverticulum

Correct Answer ( A ) Explanation: Achalasia is the most common dysmotility disorder producing dysphagia. Its exact etiology is unknown, but it is thought to be due to loss of Auerbach's plexus in the esophagus. It is associated with impaired swallowing-induced relaxation of the lower sphincter and the absence of esophageal peristalsis. Onset is usually between 20 and 40 years of age. All other answer choices are classified as structural or obstructive disorders. Esophageal webs (B) are thin structures of mucosa and submucosa located most commonly in the proximal or middle esophagus. They are associated with Plummer-Vinson syndrome and iron deficiency anemia. Schatzki ring (C) is the most common cause of intermittent dysphagia with solids. It is due to the development of a fibrous stricture near the gastroesophageal junction. It is associated with "steakhouse syndrome"—food impaction in the esophagus due to poorly chewed meat. Zenker diverticulum (D) is a progressive out-pouching of the esophageal mucosa just above the upper esophageal sphincter. It most commonly occurs in the elderly population.

A 24-year old woman presents with URI symptoms. She is 32 weeks pregnant. As part of her work-up, you order a urinalysis, which shows 2+ bacteria with no WBCs. Two days later, the lab calls you and informs you that the urine culture is positive. You call the patient back and she denies symptoms of urinary tract infection. With regards to the urine culture results, what treatment is indicated? Cephalexin 500 mg QID for 7 days Ciprofloxacin 500 mg QID for 7 days No treatment is necessary Trimethoprim-sulfamethoxazole 1 DS tablet BID for 3 days

Correct Answer ( A ) Explanation: The patient has asymptomatic bacteriuria of pregnancy confirmed by a positive urine culture and should be treated with an oral antibiotic that is known to be safe in pregnancy, such as cephalexin 500 mg QID for 7 days. Asymptomatic bacteriuria is common in the general population and in most scenarios does not require therapy. However due to the high risk of complication seen during pregnancy, it should be treated with antibiotics. It is seen in 2-10% of pregnant women and is commonly due to E. coli. Pregnant women have an increased risk of developing urinary tract infections due to the pressure that the enlarged uterus exerts on the ureters and bladder, incomplete emptying during voiding and impaired ureteral peristalsis from progesterone-induced relaxation of the ureteral smooth muscle. Complications of untreated asymptomatic bacteriuria include development of a lower urinary tract infection, pyelonephritis, renal abscess, renal failure, bacteremia, sepsis, intrauterine growth retardation, premature labor and neonatal death. Treatment options generally include cephalosporins, such as cephalexin, amoxicillin (or amoxicillin-clavulanate) and nitrofurantoin. All of which are recognized as Category B by the Food and Drug Administration; meaning that animal studies have failed to show a risk to the fetus. Treatment duration should be for 7-10 days. Ciprofloxacin (B) and trimethoprim-sulfamethoxazole (D) are Category C and D, respectively, and therefore should be avoided in pregnancy when possible. Because there is increased risk for complication during pregnancy, antibiotic treatment (C) is recommended.

A 16-year-old boy presents to the Emergency Department with a complaint of a two-day history of diffuse abdominal pain, nausea, vomiting, polydipsia, polyuria, and weakness. His blood pressure is 120/80 mm Hg, heart rate is 110, respiratory rate is 22, and temperature is 36.8°C. Initial laboratory studies reveal sodium 132, potassium 5.4, and glucose 500. Arterial blood gas demonstrates pH of 7.0, bicarbonate of 9 and partial pressure carbon dioxide of 35. What are the corrected values of serum potassium and serum sodium? Potassium 3.0 and sodium 138 Potassium 3.0 and sodium 148 Potassium 7.0 and sodium 138 Potassium 7.0 and sodium 148

Correct Answer ( A ) Explanation: The patient most likely has diabetes mellitus type 1 and is in diabetic ketoacidosis (DKA). DKA causes profound electrolyte imbalance through osmotic diuresis and poor oral intake and vomiting. Glucose draws water and electrolytes from the circulation into the urine. At the same time, elevated serum glucose causes water to flow into the vessels in an attempt to decrease the osmolar gradient caused by marked hyperglycemia. This creates dilutional hyponatremia. The true serum sodium value can be calculated by adding 1.6 mEq/L to the sodium value for every 100 mg/dL of glucose above 100 mg/dL. For this scenario, the corrected serum sodium is 132 + 1.6 x ((500-100)/100)) = 138.4 or 138. Acidosis in DKA causes potassium, magnesium, and phosphorus shift from the intracellular to the extracellular space. In addition, profound dehydration causes hemoconcentration and thus falsely elevates initial values of potassium, magnesium, and phosphorus. The corrected serum potassium in acidosis can be determined by subtracting 0.6 mEq/L from the initial potassium value for every 0.1 decrease in pH from 7.4. The corrected serum potassium: 5.4 - 0.6 x ((7.4-7.0)/0.1)) = 3.0. Every patient in DKA warrants careful monitoring and replacement of serum electrolytes. Potassium 3.0 and sodium 148 (B) is incorrect because the sodium level is overcorrected.

Which of the following characteristics of transfusion-related acute lung injury (TRALI) differentiates it from transfusion-associated circulatory overload (TACO)? High fever Hypertension Hypoxemia Pulmonary edema

Correct Answer ( A ) Explanation: Transfusion-related acute lung injury (TRALI) complicates transfusions in 1 of every 5,000 units of PRBCs. Although rare, it is associated with high mortality. The pathophysiology is thought to be due to an acute neutrophilic response that leads to endothelial damage and massive capillary leak in the pulmonary vasculature. The diagnosis is clinical with patients presenting with signs and symptoms of acute lung injury or acute respiratory distress syndrome. TRALI is characterized by hypoxemia (oxygen saturation <90% on room air), and bilateral pulmonary infiltrates presenting within 6 hours of blood transfusion. There should not be any evidence of pulmonary artery hypertension or generalized fluid overload. Hypotension, tachycardia, and fever are common. Fever is not common in transfusion-associated circulatory overload (TACO). Treatment of TRALI is supportive, with low tidal volumes for mechanical ventilation and maintenance of euvolemia. Despite the initial appearance of fluid overload, these patients are normovolemic or intravascularly depleted secondary to the increase in vascular permeability and extravasation. Diuresis may lead to further deterioration and shock. TRALI is generally characterized by hypotension not hypertension (B). Similar to TRALI is transfusion-associated circulatory overload (TACO). TACO also presents with hypoxemia (C), respiratory distress and pulmonary edema (D) on chest X-ray. However, hypertension is typical and high fever is unlikely. TACO is also associated with distended neck veins and other markers of generalized volume overload. Treatment of TACO consists of aggressive diuresis and respiratory support.

A patient is diagnosed with multiple sclerosis. Imaging shows that the majority of plaques are located in the cerebellum. Examination is positive for dysmetria and ataxia. Which of the following types of tremor would most commonly occur in this patient? Intention Pill-rolling Postural Rubral

Correct Answer ( A ) Explanation: Tremor is classified into three main types: postural, intention, and resting. An intention tremor is a tremor that occurs during movement that increases as a target is approached. The tremor is associated with cerebellar signs. The tremor is not seen at rest. Causes of intention tremor include cerebellar or brain stem disease, Wilson's disease, and drug toxicity. Pill-rolling tremor (B) is a type of resting tremor that may affect the fingers, hands, forearms, or feet and has a frequency of 4-6 Hz. Causes of resting tremor include basal ganglia lesions, parkinsonism, Wilson disease, heavy metal poisoning, hyperthyroidism, anxiety, drug withdrawal or toxicity, and benign hereditary tremor. Rubral tremor (D) is rare, occurring mainly with unusual infarcts of the midbrain's red nucleus. It occurs during rest, with movement against gravity (posture) and with intention. Postural tremor (C) occurs with sustained posture of an extremity. Symptoms increase with emotional stress or sleep deprivation and are not present during sleep. They also increase with use of TCAs, valproic acid, lithium, and bronchodilators.

A previously healthy 63-year-old woman presents with vertigo that she first noted when she woke up this morning. Throughout the day, she has experienced intense spinning that calms down with movement. She has been nauseated and having nonbloody, nonbilious emesis. On exam, she has a horizontal nystagmus beating towards the left that is exacerbated by changing head position which resolves after a few moments. She is otherwise neurologically intact and has an unremarkable head and neck exam. Which of the following findings most strongly indicates this is a peripheral nervous system lesion? Lack of other neurologic deficits Nausea Nystagmus Sudden symptom onset

Correct Answer ( A ) Explanation: Vertigo is a challenging chief complaint that can stem from the central nervous system or the peripheral nervous system. Vertigo is a perception of movement when there is none. Central vertigo is most often due to a cerebrovascular accident involving the posterior fossa. Peripheral vertigo is most often due to cranial nerve VIII or vestibular lesions. Differentiating between central and peripheral lesions is critical as the treatment pathways are quite different. A lack of other neurologic deficits (A) points is most indicative of a peripheral nervous system lesion in this patient. Nausea (B) can occur in either a central nervous system or peripheral nervous system lesion. It is often the most distressing symptom for patients with vertigo. Nystagmus (C) is associated with both central and peripheral vertigo. Isolated vertical nystagmus indicates a central process, but both peripheral and central lesions can lead to horizontal or rotary nystagmus. Sudden symptom onset (D) can be indicative of either a peripheral or central process.

A teenager presents with eye pain and redness but no pruritis. He reports this began acutely yesterday when he woke up. He describes some crusting which made it difficult for him to open his eyes. He plays football, and frequently has helmet contact, but he denies any loss of consciousness or specific orbital trauma. Interestingly, he does report most of the team has been sick since their last road trip. Examination reveals thick, boggy conjunctiva with purulent discharge. Which of the following is the most likely etiology of his condition? Allergic reaction Bacteria Chemical Virus

Correct Answer ( B ) Explanation: Conjunctivitis, a catch-all term that refers to inflammation of the conjunctiva and the small conjunctival vessels, is mostly caused by viral or bacterial etiologies. Bacterial conjunctivitis is typified by acute onset, minimal pain, minimal pruritis, purulent discharge and thick, boggy conjunctiva (chemosis). An exposure history is more common in bacterial cases. Corneal surface diseases, such as keratitis or abrasion, will likely lead to the bacterial type over other types of conjunctivitis. Allergic (A) conjunctivitis is characterized by acute or subacute onset, no pain and no exposure history. Severe pruritis is the major differentiating symptom. There is no chemical exposure given, as such, chemical (C) conjunctivitis is unlikely. Viral (D) conjunctivitis is characterized by subacute onset, minimal pain, moderate pruritis, watery discharge, photophobia and foreign-body sensation. Lymphadenopathy and prior upper respiratory infection are also common. Furthermore, a self-innoculation history, in which one eye is affected, then improves, then the other eye becomes affected, is more common in the viral type.

A 3-year-old boy is referred to the Emergency Department due to concern for Kawasaki Disease. He has persistent high fevers for five days. He also has a generalized scarlatiniform rash, red and cracked lips, and three days of watery diarrhea. Today he developed bilateral conjunctival injection without exudate. In addition to the features described, generalized lymphadenopathy is also noted. Which of this patient's clinical features is inconsistent with a diagnosis of Kawasaki Disease? Diarrhea Generalized lymphadenopathy Non-exudative conjunctivitis Scarlatiniform rash

Correct Answer ( B ) Explanation: Generalized lymphadenopathy is not typical of Kawasaki Disease and should prompt consideration of alternative infectious, rheumatologic, or oncologic etiologies. The lymphadenopathy of Kawasaki Disease is typically confined to the anterior cervical region and is almost always unilateral. At least one node must measure > 1.5 cm, although a conglomeration of nodes may be noted on ultrasound. The involved nodes do not suppurate. Notably, lymphadenopathy is the least commonly fulfilled clinical criteria in Kawasaki Disease and is absent in up to 75% of cases. Bilateral non-exudative conjunctivitis (C) is a classic feature of Kawasaki Disease. The conjunctivitis also classically spares the limbus, creating a halo around the iris. Anterior uveitis may also be present. Unilateral conjunctivitis or exudate should prompt investigation for alternative diagnoses. Diarrhea (A), along with other gastrointestinal symptoms such as abdominal pain and vomiting, is common in Kawasaki Disease, but it is non-specific and non-diagnostic. A scarlatiniform rash (D) is also commonly associated with Kawasaki Disease. As noted above, the rash of Kawasaki Disease is classically morbilliform but may also be macular, maculopapular, or scarlatiniform and occasionally may mimic erythema multiforme. Vesicular or bullous lesions should prompt evaluation for alternative etiologies. Notably, the rash of Kawasaki Disease often begins as perianal or perineal desquamation. However, the classic periungal desquamation does not typically occur until the convalescent phase of the disease. Inflammation of the Bacille Calmette-Guerin (BCG) vaccine site is also associated with Kawasaki Disease, but this is often not observed in the United States due to lack of routine BCG vaccination.

Which of the following is most characteristic of phimosis? Collection of dilated and tortuous veins surrounding the spermatic cord Inability to retract foreskin over glans Inability to return retracted foreskin over glans Painless cyst filled with sperm

Correct Answer ( B ) Explanation: Phimosis is the inability to retract foreskin over the glans penis. It is a complication seen in uncircumcised men. About 50% of boys typically are able to retract their foreskin by 1 year of age and 80% by age 3. Topical estrogen therapy has been reported as successful, but no randomized trials support its use. However, low-potency topical corticosteroid therapy combined with daily prepuce retraction appears effective for phimosis. If retraction of foreskin is unsuccessful, the patient will require circumcision. A varicocele (A) is a collection of dilated and tortuous veins surrounding the spermatic cord. It is most often asymptomatic. Paraphimosis (C) is the inability to return retracted foreskin over the glans. Because vascular engorgement leads to necrosis of the glans, acute paraphimosis needs urgent medical attention. A dorsal slit procedure may be required if other reduction techniques are unsuccessful. Ice packs and plastic wrap may reduce the edema enough to allow manual reduction. A painless cyst filled with sperm (D) is called a spermatocele and can be palpated distinct from the testis. They transilluminate, are generally of no consequence, and do not affect fertility.

A five-year-old boy presents to the emergency department with nasal bleeding lasting greater than two hours. In the emergency department, the nurse applied pressure for 20 minutes without successful resolution. The patient appears well and has a normal heart rate and blood pressure. The patient's mother reports that the child's father has a bleeding disorder for which he occasionally uses a nasal spray. The patient has never had testing, and this is his first physician encounter for bleeding. Which lab abnormality would you expect to find for the child's inherited condition? Decreased platelet count Increased bleeding time Increased platelet count Increased prothrombin time

Correct Answer ( B ) Explanation: The condition described is von Willebrand disease (VWD). It is the most common hereditary bleeding disorder. The most common type, type I, is inherited in an autosomal dominant fashion. Increased tendency to bleed is the result of a qualitative or quantitative defect in von Willebrand factor (VWF), a protein necessary for platelet adhesion function. The resultant platelet type bleeding commonly affects mucosa leading to dental and nasal bleeding as well as menorrhagia and increased bleeding after delivery of a child. Testing for VWD requires examination of the blood for both quantitative and qualitative deficiencies of VWF. VWF antigen assay evaluates quantity of von Willebrand factor. The level of functionality of VWF is tested with one of the following: glycoprotein binding assay, collagen binding assay, ristocetin cofactor activity (RiCof) or a ristocetin-induced platelet agglutination (RIPA) assay. Bleeding time is increased. PTT is normal or may be increased. Platelet counts (A and C) are incorrect since it is the function, not the number, of platelets that is adversely impacted in von Willebrand disease. The platelets have abnormalities in a protein (VWF) required for adhesion function. Partial thromboplastin time (PTT) not prothrombin (PT) (D) is increased because factor VIII is bound to von Willebrand factor. Accordingly, a decrease in VWF will also result in a decrease in factor VIII, which explains the increased PTT. Reduced factor VIII has no corresponding change in prothrombin time (PT).

Which of the following infectious agents is associated with reactive arthritis? Adenovirus Chlamydia trachomatis Staphylococcus aureus Streptococcus pyogenes

Correct Answer ( B ) Explanation: The development of reactive arthritis is associated with a number of bacterial agents including Chlamydia trachomatis. Reactive arthritis is characterized by oligoarthritis, urethritis and uveitis - "can't see, can't pee, can't climb a tree." The syndrome occurs in susceptible hosts after infection with certain agents including C. trachomatis. The arthritis typically develops 2-6 weeks after an episode of urethritis or dysentery. The most commonly involved joints are the ankles and knees and the syndrome typically involves bilateral joints. Conjunctivitis may be present early and progress to uveitis later in the disease process. Management focuses on decreased inflammation with non-steroidal anti-inflammatory medications. C. trachomatis-triggered reactive arthritis improves faster with antibiotics. Adenovirus (A) is a common virus that can cause conjunctivitis. Staphylococcus aureus (C) exotoxin can cause staphylococcal scalded skin syndrome (SSSS) and toxic shock syndrome (TSS) but is not associated with reactive arthritis. Streptococcus pyogenes (D) exotoxin has also been implicated in TSS but not in reactive arthritis.

A 29-year-old woman presents to the office complaining of fever, rash, and arthralgias that began after starting penicillin for a dental infection. The physical exam is unremarkable. A complete blood count reveals the presence of peripheral eosinophilia. The urine contains red cells (no red cell casts) and white cell casts, but no bacteria. Serum BUN is 20 mg/dL, and creatinine is elevated at 2 mg/dL (baseline 1 mg/dL). Which of the following is the most likely diagnoses? Glomerulonephritis Interstitial nephritis Prerenal azotemia Urinary tract infection

Correct Answer ( B ) Explanation: The patient has developed renal insufficiency (her serum creatinine has doubled). The active urinary sediment suggests that her acute kidney injury is the result of intrinsic renal disease (e.g., acute tubular necrosis (ATN), acute glomerulonephritis, or acute interstitial nephritis). Of these, acute interstitial nephritis is most likely given the presence of fever (seen in >80% of patients), rash (seen in 25-50% of patients), and peripheral eosinophilia (80%). Acute interstitial nephritis results from interstitial inflammation, most commonly in response to medication (penicillin in this case), but it is also associated with infections and autoimmune disease. The most commonly implicated medications include: penicillins, diuretics, anticoagulants, and NSAIDs. Treatment is mainly supportive, but it is critical to discontinue the offending agent. Steroids may be necessary if there is significant renal impairment. Renal function generally returns to baseline over weeks. Glomerulonephritis (A) is associated with dependent edema and hypertension, neither of which was mentioned. Additionally, glomerulonephritis is characterized by a serum BUN:Cr ratio of >20:1 and by the presence of red cell casts. Prerenal azotemia (C) results from decreased renal perfusion due to a decrease in intravascular volume, a fall in cardiac output, or a change in vascular resistance. Nothing in the fact pattern suggests any of these conditions. Additionally, prerenal azotemia is associated with a BUN:Cr ratio of >20:1. An isolated urinary tract infection (D) typically does not cause acute renal insufficiency. While the patient is febrile, there are no signs or symptoms to suggest a UTI.

A 45-year-old woman presents to the ED with the rash seen above. Which of the following is true regarding this condition? Always appears in a dermatomal distribution Firm stroking of the skin produces a wheal Mucous membrane involvement is commonly seen The rash is a mild form of Stevens-Johnson syndrome

Correct Answer ( B ) Explanation: This is an urticarial rash (hives). Urticaria may occur from a variety of agents, including foods (lobster, strawberry); drugs (penicillin, aspirin); infections (hepatitis, mononucleosis, coxsackie); change of temperature; and connective tissue disorders, to name a few. It can occur in isolation or as part of a systemic anaphylactic reaction. The characteristic rash appears as edematous plaques with pale centers and red borders. The rash of urticaria is transient, lasting less than 24 hours, though new lesions may continuously develop. Dermatographism is universally present and results in development of an urticarial wheal within 30 minutes of firm skin stroking. Herpes zoster (shingles) (A) always occurs in a dermatomal distribution and is characterized by a vesicular rash on an erythematous base. Mucous membrane involvement is pathognomonic of Stevens-Johnson syndrome (C and D) and may also be seen with anaphylaxis but is not typically seen with cases of simple urticaria.

A 17-year-old boy presents to the ED with a painless mass on the left side of his neck, which he first noticed three months ago, that is now increasing in size. The patient also reports a 10-pound weight loss and night sweats. On exam, you note a 5 cm rubbery, firm, and mobile mass located in the anterior cervical triangle. The overlying skin is intact, with no redness, streaking, or drainage. What is the most likely diagnosis? Burkitt lymphoma Hodgkin's lymphoma Non-Hodgkin's lymphoma T-cell lymphoma

Correct Answer ( B ) Explanation: This patient most likely has Hodgkin's lymphoma. Lymphoma is the most common malignancy of the head and neck in children. Hodgkin's lymphoma typically occurs in teenagers, with a neck mass serving as the presenting complaint 80% of the time. The mass is most often supraclavicular or in the anterior cervical triangle. It is often accompanied by constitutional symptoms such as weight loss, fevers, and night sweats (B symptoms). Non-Hodgkin's lymphoma (C) is most common in children between two and twelve years of age; extranodal involvement as well as lymphadenopathy below the clavicles are frequently seen. Burkitt lymphoma (A) is a form of non-Hodgkin's lymphoma. It is considered the fastest growing human tumor and unlikely to grow slowly over the course of four months. Although neck masses can occur, in the United States the intra-abdominal form of the disease is most common. There are four types of T-cell lymphoma (D). Collectively, they comprise about 10% of non-Hodgkin's lymphomas and are much less likely to be the cause of this patient's illness.

A 39-year-old woman presents with pain and swelling around the eye as seen above. Extraocular motions are intact and vision is normal. Which of the following is the most appropriate management? CT scan of the face Oral and topical ocular antibiotics Topical antibiotics Warm compresses and ophthalmology follow up

Correct Answer ( B ) Explanation: This patient presents with dacrocystitis and should be treated with topical ocular and oral anti-staphylococcal antibiotics. Dacrocystitis is an acute infection of the lacrimal sac secondary to lacrimal duct obstruction. It is typically caused by Staphylococcus aureus. Symptoms include swelling, redness, pain and tenderness to palpation over the lacrimal sac. In addition to topical and oral antibiotics, patients should be treated with warm compresses and gentle massage of the area. Complications of improperly treated dacrocystitis include periorbital and orbital cellulitis. CT scan of the face (A) is unnecessary unless a deeper infection is suspected. Topical antibiotics alone (C) are inadequate for treatment. Warm compresses (D) can aid in treatment but are inadequate for resolution without antibiotics.

A previously healthy 25-year-old woman presents to your office with complaints of flank pain and burning with urination. She tells you that two days ago she started experiencing symptoms that also included frequent urination, nausea, vomiting and chills. Her temperature is 102°F and she has costovertebral angle tenderness on physical exam. Which of the following is the most appropriate therapy? Amoxicillin-clavulanate Ciprofloxacin Fosfomycin Nitrofurantoin

Correct Answer ( B ) Explanation: Urinary tract infections (UTIs) are differentiated into two types. Cystitis is an infection of the bladder and pyelonephritis is an infection of the kidneys. Pyelonephritis is a more serious infection than cystitis and the antibiotics to treat the infection are different than those used with cystitis. Patients present with symptoms similar to cystitis such as dysuria, urgency, and urinary frequency, but also have fever, chills, nausea, vomiting, flank pain and costovertebral angle tenderness. Mild to moderate uncomplicated pyelonephritis may be managed on an outpatient basis. Fluoroquinolones such as ciprofloxacin or levofloxacin are commonly used to treat outpatient pyelonephritis. Another option is trimethoprim-sulfamethoxazole. Patients considered to have complicated pyelonephritis including pregnant women, diabetics, immunocompromised, and those with renal failure or hospital acquired infection should be treated in the inpatient setting with parenteral antibiotics. Amoxicillin-clavulanate (A) is a beta lactam that may be used in the treatment of cystitis. Oral beta lactam agents are less effective than other agents for treatment of pyelonephritis. If the pathogen is susceptible and an oral beta lactam agent is continued, it should be administered for at least 14 days. Fosfomycin (C) and nitrofurantoin (D) are also used in treatment of cystitis, but should be avoided in pyelonephritis as they do not achieve sufficient renal tissue levels.

A 21-year-old woman presents to your office for her annual exam. While taking the patient's history, she reveals that she has been eating excessive amounts of food and then vomiting almost daily for the past six months. She has started working with a nutritionist and is seeing a cognitive behavioral therapist but would like to know if there are any medications that might help her treatment. Which of the following is the most appropriate medication for this patient? Desipramine Diazepam Fluoxetine Lorazepam

Correct Answer ( C ) Explanation: Bulimia nervosa is an eating disorder characterized by recurrent episodes of binge eating and inappropriate compensatory behavior such as self-induced vomiting or laxative abuse. For diagnosis, the behaviors must occur at least once per week for 3 months. Individuals feel out of control with their behaviors and are overly concerned with their physical appearance and weight. Treatment for bulimia nervosa includes psychotherapy, nutritional rehabilitation, pharmacotherapy and management of medical complications. The selective serotonin reuptake inhibitor (SSRI) fluoxetine is the only SSRI approved for use in treating bulimia nervosa. The tricyclic antidepressant desipramine (A) is a third-line option for the treatment of bulimia nervosa. Diazepam (B) and lorazepam (D) are benzodiazepines, which have no role in the treatment of bulimia nervosa.

An 81-year-old man presents to the Emergency Department in respiratory distress. He is sitting upright and appears anxious, dyspneic and diaphoretic. Vital signs show blood pressure of 190/110 mm Hg, heart rate of 130 beats/minute, respiratory rate of 35 breaths/minute, and oxygen saturation of 85% on room air. Which of the following physical examination findings most strongly suggests heart failure as the cause of his respiratory distress? Jugular venous distension Presence of a fourth heart sound Presence of a third heart sound Wheezing

Correct Answer ( C ) Explanation: Respiratory distress can result from numerous pathologic states, including obstructive airway disease (e.g. asthma and chronic obstructive pulmonary disease), decompensated heart failure, myocardial infarction, pneumonia, upper airway obstruction, tension pneumothorax, pulmonary embolism, fat embolism, and neuromuscular disease, among others. Emergency physicians must quickly determine the cause of respiratory distress in order to initiate appropriate treatment. Heart failure is a common cause of respiratory distress. A weakened or diseased left ventricle or one facing high systemic pressures cannot adequately pump blood and as a result blood pools in the lungs, leading to pulmonary edema and clinical symptoms of congestive heart failure. Symptoms include dyspnea on exertion, dyspnea at rest, orthopnea, paroxysmal nocturnal dyspnea, and peripheral edema. However, these symptoms are seen in many conditions and cannot be used to distinguish congestive heart failure from other causes of dyspnea. Physical exam findings that suggest congestive heart failure include presence of a third heart sound or S3 gallop (likelihood ratio (LR) 11.0), hepatojugular reflex (LR 6.4), and jugular venous distention (LR 5.1). The combination of an S3 gallop and a chest radiograph showing pulmonary venous congestion or interstitial edema is highly suggestive of congestive heart failure. Jugular venous distention (A) is seen in congestive heart failure, but has a lower likelihood ratio than an S3 gallop. Presence of a fourth heart sound or an S4 (C) and wheezing (D) are not predictive of congestive heart failure.

A healthy 6-year-old boy presents to the ED with bloody diarrhea. He was in his usual state of health until one week ago when loose, watery stools (up to 10 per day) were noted. He was seen by his pediatrician four days ago but has since developed increasing amounts of blood and pus in his stools along with a low-grade fever. Mom states there is no recent travel, antibiotic use, or known sick contacts. His vitals are heart rate 118 beats per minute, oxygen saturation 100% on room air, and rectal temperature of 38.3°C. Your physical exam reveals a mildly tender abdomen without localization, rebound, guarding, or peritoneal signs. You note grossly bloody stool on rectal exam. A brief discussion with his pediatrician confirms your suspicion of an invasive bacterial diarrhea; a stool culture was positive for Shigella. Which of the following statements is true regarding this condition? Antibiotics should be avoided because this is a severe case and the patient is at highest risk of developing hemolytic uremic syndrome Antidiarrheal agents (such as diphenoxylate and atropine) are indicated, given the frequency of loose stools Extraintestinal manifestations such as hallucinations, confusion, and seizures may occur Oral rehydration should be avoided; IV fluids should be initiated

Correct Answer ( C ) Explanation: Shigella species cause an invasive diarrhea that rarely infects infants younger than three months old and is most common between two and three years of age. Infection is typically transmitted by person-to-person (fecal-oral) contact or through ingestion of contaminated material. Clinical illness varies from mild to severe, with some patients exhibiting abdominal cramps and tenderness. Dysentery (diarrhea with significant blood, pus, and mucus) occurs in approximately 33% of patients. Some patients may also develop extraintestinal manifestations such as reactive arthritis, seizures, and hallucinations. Although antibiotics (A) are generally not indicated for the treatment of mild illness, they should be considered in more severe cases. Hemolytic uremic syndrome (HUS) is a potentially fatal complication that develops when antibiotics are administered to children (but not adults) with enterohemorrhagic E. coli (O157.H7). HUS results from the release of Shigella-like toxin by dying E. coli (not Shigella itself as which occurs from Shiga toxin). For this reason, it is recommended that empiric antibiotic therapy be held in pediatric patients with infectious diarrhea pending culture results. In this case, the causative organism is already known, and antibiotic therapy can be safely administered. Antidiarrheal agents (B) are not recommended because they can worsen bacterial invasion of the bowel wall and prolong the infection and carrier state. If dehydration is a concern, oral rehydration (D) should be attempted prior to initiating IV fluids because the patient is not vomiting and is able to tolerate oral intake.

A 46-year-old man presents with pain in his submandibular space for 48 hours that worsens when he eats. He denies current fever or chills, but endorses "feeling dehydrated" due a recent febrile illness. Exam shows erythema and swelling of the submandibular gland and pus is easily expressed. He is handling secretions normally and phonation is unimpaired. Which of the following is the most likely cause of his symptoms? Ludwig angina Salivary gland tumor Sialadenitis Sialolithiasis

Correct Answer ( C ) Explanation: Sialadenitis typically presents with acute swelling of either the parotid or submandibular glands, as well as pain that increases during meals as the secretion of saliva during mastication irritates the inflamed tissue. Diagnosis is clinical, as the physical exam will show a tender, erythematous duct opening with pus classically being expressed with massage. Sialadenitis is usually caused by S. aureus and frequently occurs in the settings of chronic illness or recent dehydration. Treatment initially requires intravenous antibiotics with transition to an oral antistaphylococcal agent based on improvement. A 10-day course of oral antibiotics is standard. Measures should be taken to encourage salivary flow by maintaining appropriate hydration, massaging the affected gland, and using warm compresses and sialagogues such as lemon drops. If patients fail to have full resolution of pain in 2-3 weeks, further diagnostic testing with either ultrasound or CT scan may be needed to find an underlying cause such as strictures, abscess formation, or obstructing tumor or stone formation. Ludwig angina (A) presents with edema and erythema of the external upper neck under the chin and floor of the mouth. The exam may show an upwardly displaced tongue as cellulitis and pus gather in the floor of the mouth. Phonation is muffled and tongue protrusion is typically impaired as a result. Salivary gland tumors (B) are more likely to occur in the parotid glands as opposed to the submandibular glands. They are typically noted as small, asymptomatic masses which may be present for years before being detected. Sialolithiasis (D), or calculus formation in the salivary glands, is most common in Wharton duct, which drains the submandibular glands. While sialolithiasis also presents with post-prandial pain and local swelling, it is more likely to present with a palpable stone, particularly in Wharton duct, and less likely to allow expression of pus on exam.

Which of the following correctly matches the stage with the clinical finding in a patient with chronic lymphocytic leukemia (CLL)? Stage 0 - No findings, just family history Stage 1 - Lymphocytosis only Stage 2 - Lymphocytosis and splenomegaly with no anemia Stage 3 - Lymphocytosis, splenomegaly, thrombocytopenia and hemoglobin >15 g/dL

Correct Answer ( C ) Explanation: Stage 2 chronic lymphocytic leukemia (CLL) is classified as lymphocytosis, plus or minus lymphadenopathy, splenomegaly with no anemia. CLL is a neoplasm characterized by the accumulation of monoclonal lymphocytes of B-cell origin. The cells accumulate in the bone marrow, lymph nodes, liver, spleen, and occasionally other organs. CLL is the most common leukemia (one third of all cases) in the Western world and is twice as common as CML. The disease occurs rarely in those younger than 30 years; most patients with CLL are older than 60 years. CLL increases in incidence exponentially with time; by age 80, the incidence rate is 20 cases per 100,000 persons per year. The male-female ratio is approximately 2:1. Most patients with CLL are asymptomatic, and the disease is diagnosed when absolute lymphocytosis is noted in the peripheral blood during evaluation for other illnesses or when the patient undergoes a routine physical examination. Symptoms such as fatigue, lethargy, loss of appetite, weight loss, and reduced exercise tolerance are nonspecific. CLL is characterized by absolute lymphocyte counts that typically range from 5000 to 600,000/µL in the peripheral blood. Even with markedly elevated WBC counts, hyperviscosity symptoms rarely occur, probably because of the small size and pliability of the cells. Anemia (hemoglobin <11 g/dL) is present in 15 to 20% of patients at diagnosis, and thrombocytopenia (platelet count <100,000/µL) in 10%. However, bone marrow replacement and hypersplenism, which are seen with progressive disease, increase the frequency of anemia and thrombocytopenia. The anemia is usually normochromic and normocytic, and the reticulocyte count is normal unless the patient has autoimmune hemolytic anemia. Stage 0 (A) is when patient just has lymphocytosis. Stage 1 (B) patients have both lymphocytosis and lymphadenopathy. Stage 3 (D) patients have lymphocytosis, plus or minus lymphadenopathy, splenomegaly, no anemia and no thrombocytopenia.

Which of the following constitutes the Systemic Inflammatory Response Syndrome (SIRS) criteria? Heart rate >80 and Respiratory Rate > 16 Heart rate >90 Heart rate >90 and temperature >100.4°F Temperature >100.4°F and WBC >10,000

Correct Answer ( C ) Explanation: The SIRS criteria is part of the sepsis syndrome spectrum of diseases. SIRS often represents the body's host response to an infection. Although research has found that SIRS criteria alone does not predict an increased mortality, it should prompt continued investigation for an underlying pathology. The presence of organ dysfunction and shock, however are significant predictors of adverse outcomes and should be fully addressed. Sepsis is the tenth most common cause of death in the US. A heart rate >90 (B) is one of the SIRS criteria but at least two of these need to be present to qualify as SIRS. The heart rate must be >90 and the respiratory rate >20 (A) to meet SIRS criteria. The temperature is correct, however, a WBC >12,000 represents one of the SIRS criteria, not >10,000 (D).

A 16-year-old girl is in your clinic with her mother for a sports physical examination. She plays volleyball and denies chest pain, difficulty of breathing, or family history of heart disease. She started a vegan diet six months ago. She is a straight A student. Physical exam is normal. You obtain a complete blood count, which reveals hemoglobin of 10 g/dL and mean corpuscular volume (MCV) of 110 fL. Which of the following is the most likely diagnosis? Iron deficiency anemia Lead toxicity Vitamin B12 deficiency Zinc deficiency

Correct Answer ( C ) Explanation: The girl has macrocytic anemia, which is defined by a decreased hemoglobin and MCV > 100 fL. The macrocytic anemia is most likely secondary to a vitamin B12 deficiency. Vitamin B12 deficiency can result from inadequate dietary intake in certain settings like strict vegans who avoid all animal products. Animal products (meat and dairy products) provide the only dietary source of cobalamin for humans. The usual Western diet contains 5 to 7 mcg of cobalamin per day, while the minimum daily requirement is 6 to 9 mcg per day. Malabsorption of vitamin B12 also occurs in pernicious anemia due to intrinsic factor deficiency, and in ileal resections and Crohn disease. Breast-feeding infants of vitamin B12-deficient mothers are also at risk for significant deficiency. Vitamin B12 deficiency should be suspected in patients with one or more of the following clinical or laboratory findings: oval macrocytic red blood cells (MCV >100 fL) on the peripheral blood smear, the presence of hypersegmented neutrophils on the peripheral blood smear, and unexplained neurologic signs and symptoms. The common neurologic symptoms in children include irritability, hypotonia, developmental delay, developmental regression, and involuntary movements. On the other hand, sensory deficits, paresthesias, and peripheral neuritis are seen in adults. Iron deficiency anemia (A) and lead toxicity (B) both present with microcytic anemia wherein MCV is less than 80 fL. Zinc deficiency (D) presents with alopecia, delayed wound healing and dermatitis. Zinc deficiency does not present with abnormalities in the CBC.

You receive the laboratory report of a pericardial effusion sample sent yesterday from an inpatient with metastatic lung cancer. Which of the following results would you most expect? Exudate with Gram-positive bacilli Exudate with low-protein and high glucose levels Exudate with positive cytology Transudate with elevated carcinoembryonic antigen levels

Correct Answer ( C ) Explanation: There are several different reasons why the pericardial sac fills with fluid. Pericardial effusion often poses several diagnostic questions. In order to work through a differential, a sample of pericardial fluid can be sent to the lab for analysis testing. As is common with other fluid analyses, the first step in evaluating pericardial fluid is to differentiate transudate from exudate. Transudate represents an imbalance between vessel hydrostatic and oncotic pressure. Transudates are usually associated with some cardiac disease, such as congestive heart failure, or hepatic disease, such as cirrhosis. Other causes of transudative effusion include nephrotic syndrome, hypothyroidism and amyloidosis. On the other hand, exudates herald the presence of some traumatic injury or inflammation. Exudate can be infectious in nature, as in viral, bacterial or fungal pericarditis, or even myocarditis and endocarditis. Exudates are also commonly associated with autoimmune rheumatic conditions, such as rheumatoid arthritis or systemic lupus erythematosus. Cancer, either primary or metastatic, can also produce a pericardial exudate. In addition, exudates can be bloody, as in bleeding disorders or direct trauma. Once the pericardial effusion is deemed exudative, other tests are employed. Total cell counts, WBC differentials, fluid glucose, total protein and lactate dehydrogenase levels, microscopic examination, Gram stain, culture and susceptibility testing, AFB smear and culture, cytology and parasitic testing round out the typical battery of tests used to determine the source of a pericardial exudative effusion. Light's criteria are used to help differentiate transudative from exudative effusions - most commonly in pleural effusions but the table below can also be applied to pericardial effusions. Gram-positivity and the presence of bacteria (A) suggests infectious, not neoplastic, etiology. Exudates typically have a high total protein, low glucose (B) and high lactate dehydrogenase levels. Although malignancy is associated with pericardial effusion with elevated carcinoembryonic levels, they are mostly exudative, not transudative (D).

An 80-year-old man reports 3 months of high pitched ringing in his ears. He denies head trauma, fever, change in medications or diet, gross hearing loss, dizziness or vertigo, focal neurologic deficits or vision changes. Which of the following is the most likely diagnosis? Epistaxis Meniere's disease Tinnitus Vestibular neuritis

Correct Answer ( C ) Explanation: Tinnitus is a high-pitched noise in the ear that may last for several minutes or persist. It may be described as humming, buzzing or ringing. When symptoms are severe, it may interfere with concentration or sleep. The otologic exam is often normal. Cerumen impaction is common, and head or neck carotid bruit may be present. Tinnitus is also associated with some degree of hearing loss. Evaluation should include a review of the patient's blood pressure, allergies, serum lipids and thyroid function. Since most causes are due to presbycusis (age-related, cumulative hearing damage and loss), effective therapies are limited. Medication management involves trials of diuretics, antihistamines, anticonvulsants, tricyclic antidepressants and benzodiazepines. Epistaxis (A) is the formal term for nosebleed. Meniere's disease (B) is classically described as a tetrad of symptoms: vertigo, tinnitus, hearing loss and aural fullness. This patient only has tinnitus. Vestibular neuritis (D) is sudden, often profound, prolonged vertigo without hearing impairment in an otherwise healthy patient. Tinnitus is not associated with vestibular neuritis.

Which of the following findings is classically associated with pyloric stenosis? Bilious vomiting Elevated lipase Metabolic alkalosis Poor feeding

Correct Answer ( C ) Explanation: Vomiting from pyloric stenosis classically causes a hypochloremic metabolic alkalosis. Hypertrophic pyloric stenosis is the most common cause of obstruction in the first month of life. It is more common in boys and typically presents with projectile vomiting that occurs promptly after feeds. Patients typically present 2 to 6 weeks after birth. If vomiting is prolonged, it can lead to hypochloremic metabolic alkalosis secondary to loss of fluids from the stomach. Physical examination may reveal an "olive" shaped mass representing the hypertrophied pyloric sphincter. Diagnosis is made with either abdominal ultrasound or upper GI series. Non-bilious vomiting (A) is typically seen in pyloric stenosis as the obstruction is proximal to the bile ducts entering the duodenum. An elevated lipase (B) is not seen in pyloric stenosis. Patients with pyloric stenosis typically have vigorous feeding (D).

Which of the following is the most common infection associated with erythema multiforme? Borrelia burgdorferi Haemophilus influenzae Type b Hepatitis C virus Herpes simplex virus

Correct Answer ( D ) Explanation: Although numerous infections have been reported in association with erythema multiforme (EM), herpes simplex virus (HSV) is the most common and best documented. Recurrent EM also is associated with infection with HSV. Demonstration of prior exposure to HSV by serology and documentation of a cutaneous recurrence of HSV infection was noted in a series of patients with recurrent EM and was less common in patients with a single episode. The pathogenesis of EM is incompletely understood, but evidence increasingly implicates a host-specific, cell-mediated immune response to an antigenic stimulus that targets keratinocytes at the dermal-epidermal junction. EM has variable cutaneous manifestations. EM is characterized by the acute onset of a symmetric, fixed cutaneous eruption of erythematous macules, papules, vesicles, or bullae most commonly distributed on the palms, dorsal surfaces of the hands and feet, and extensor surfaces of the arms and legs with relative sparing of the face, trunk and mucous membranes. Lesions can expand and evolve over several days to assume the classic annular "target" appearance with a dusky, necrotic center surrounded by a ring of edema and pallor and an erythematous border. Borrelia burgdorferi (A) is associated with Lyme's disease and skin condition called erythema migrans. Haemophilus influenzae Type b (B) is the etiologic agent in a variety of infections although less common with the advent of the pediatric Hib vaccine but can still be a causative agent of epiglottitis. It is typically not associated with erythema multiforme. Hepatitis C virus (C) is not a common cause of erythema multiforme although rash can be associated with the treatment of hepatitis C with telaprevir.

A mother brings her full-term 3-day-old male to the ED after she noticed an episode of dark, tarry stool at home. Mom delivered the newborn vaginally with no complications. Mom is a vegan and is breastfeeding. On exam, the patient is afebrile. His abdomen is soft with no palpable masses. Rectal exam reveals dark, tarry stool that is guaiac positive. Which of the following is the most likely cause of this patient's presentation? Meckel's diverticulum Milk protein allergy Necrotizing enterocolitis Swallowed maternal blood

Correct Answer ( D ) Explanation: In the neonatal period, the most likely cause of lower gastrointestinal bleeding is swallowed maternal blood, from either delivery or cracked nipples during breast-feeding. The Apt test differentiates fetal from maternal blood. The blood in question is mixed with alkali to detect conversion of oxyhemoglobin to hematin. Fetal hemoglobin is more resistant to denaturation than adult hemoglobin is. If the supernatant stays pink after addition of alkali, the blood is fetal in origin (a positive test). This should not be confused with the Kleihauer-Betke test, used to detect fetal-maternal hemorrhage in the pregnant female. Necrotizing enterocolitis (C) is a complication of premature infants and presents with abdominal distension, bloody stools, and feeding intolerance. Milk protein allergy (B) should be suspected after introduction of a new formula. Infants who are breastfed are not immune to this condition if Mom consumes cow's milk in her diet. Since the mother in this vignette is a vegan and is breastfeeding, it is unlikely that a milk protein allergy is responsible for the patient's guaiac-positive stool. In older infants with painless hematochezia, congenital malformations such as small intestine duplication and Meckel's diverticulum (A) should be considered. Meckel's diverticulum is most common at two years of age.

A 13-month-old girl presents with her mother who says that her daughter has been sick for a 10 days. The girl had an initial period of "head-cold symptoms" and is now coughing constantly. She is a well-nourished infant who demonstrates a rapid, consecutive cough with a high-pitched inspiratory "whoop." Records show that the family declined vaccinations. Which of the following interventions will be most helpful to expedite resolution of this patient's illness? Humidification of inspired air Nebulized albuterol Oral amoxicillin Oral clarithromycin

Correct Answer ( D ) Explanation: Oral macrolide antibiotics, using either erythromycin, azithromycin, or clarithromycin, are the first-line treatment for the respiratory infection pertussis. Pertussis is an acute respiratory illness that primarily causes illness in children under 2 years of age. Though immunization exists against the causative pathogen, Bordetella pertussis, neither immunization nor history of infection ensures lasting immunity, and adults often serve as infectious reservoirs for the disease. Pertussis generally begins with a one to two-week period of malaise, sneezing, and anorexia. It is followed by a persistent cough, with the classic, high-pitched inspiratory whoop. Posttussive emesis also suggests pertussis. Non-specific laboratory findings can include a white blood cell count of 15,000-20,000 with up to 80% lymphocytes. However, pertussis is definitively diagnosed by isolating the organism from a nasopharyngeal culture. A 4-7 day course of treatment with a macrolide antibiotic should be used to reduce coughing severity and shorten the duration of carriage. Close contacts should receive the same treatment as prophylaxis against illness. Humidification of inspired air (A) and Nebulized albuterol (B) can be used as adjunct treatments for pertussis, but a macrolide is the treatment of choice, not amoxicillin (C).

A 29-year-old woman is concerned with a rash. On inspection, you appreciate a central facial rash. You notice that the nasolabial folds are not affected. You also detect alopecia and lip crusting. The neck, chest and trunk are void of cutaneous abnormalities. The dorsal digits, and other extensor surfaces, are not affected. Which of the following is the most likely diagnosis? Dermatomyositis Psoriasis Rosacea Systemic lupus erythematosus

Correct Answer ( D ) Explanation: Rashes can be difficult to evaluate, especially when considering dermatologic versus rheumatologic conditions. A crossover occurs in four main conditions: rosacea, systemic lupus erythematosus (SLE), psoriasis and dermatomyositis. SLE is associated with a red, butterfly-shaped, malar rash, which is located on the central face but not in the nasolabial folds. In comparison, rosacea's malar rash affects the central face and nasolabial folds. Furthermore, systemic symptoms do not occur in rosacea. Facial rash can also present with dermatomyositis, however, this central facial rash is not termed malar, but rather heliotrope, and appears as a waxing-waning violet-colored (violaceous) rash that mainly affects the eyelids but also affects the nasolabial folds. To add to this difficulty, SLE is associated with another rash called discoid rash. This is described as raised chronic inflammatory sores with scarring and scaling, which occur on the face, ears and scalp, and can easily be confused with psoriasis. Patients with SLE may have a malar rash, or discoid rash, or both. Dermatomyositis (A) is associated with a shawl or cape-like rash of the neck, shoulders and upper chest, as well as knuckle skin abnormalities (Gottron's papules). Psoriasis (B) typically produces scaly-crusty lesions on extensor surfaces, like knuckles, elbows and knees. Alopecia and hemorrhagic crusting of the lips occurs more commonly in SLE than

A 67-year-old woman with sick sinus syndrome complains of significant lightheadedness. If she is driving and a "dizzy spell" begins, she has to pull off the road and wait for it to pass. You capture an ECG during one of her spells in the office and note a bradycardia in the 50s. Which of the following is the best treatment recommendation? Atenolol Defibrillator Digitalis Pacemaker

Correct Answer ( D ) Explanation: Sick sinus syndrome occurs as a result of disease of the sinoatrial (SA) node. It is associated with tachycardia-bradycardia syndrome in which the sinus rate varies from fast to slow and back again. ECG shows an irregular rhythm with pauses in sinus activity. Many patients with sick sinus syndrome respond favorably to permanent pacemaker placement. In fact, the treatment of choice for any symptomatic bradycardia that accompanies sick sinus syndrome is pacemaker placement. Atenolol (A) and other beta-blockers may cause sinoatrial arrest or block in patients with sick sinus syndrome. Digitalis (C) may cause bradydysrhythmias in patients with sick sinus syndrome. Defibrillators (B) are not recommended in the treatment of sick sinus syndrome.

A 16-year-old girl presents to the Emergency Center with complaints of vaginal discharge. Speculum exam reveals a frothy discharge and punctate hemorrhages of the cervix. What is the treatment of choice? Azithromycin Ceftriaxone Doxycycline Metronidazole

Correct Answer ( D ) Explanation: The above patient has findings of acute cervicitis as manifest by vaginal discharge and cervical changes on speculum exam. The punctate hemorrhages on the patient's cervix describe "strawberry cervix," which is a classic finding of trichmoniasis, caused by Trichomonas vaginalis. Notably, however, strawberry cervix is estimated to be present in only 2% of cases and cannot be utilized to rule out trichmoniasis. Frothy, green vaginal discharge is also considered classic for Trichomonas, but the consistency and color the discharge may vary considerably. In addition to discharge, affected patients may complain of dysuria, urinary frequency, dyspareunia, or lower abdominal pain. Others are completely asymptomatic but still contagious. As signs and symptoms are neither sensitive nor specific for diagnosis, a wet prep should be obtained. If the motile trichomonads are not noted but trichmoniasis is suspected, the specimen should be sent for culture. The treatment of choice for is metronidazole. Tinidazole is an alternative therapy. Azithromycin (A) is the treatment of choice for chlamydial cervicitis. It is given in a single one-gram dose. Ceftriaxone (B) is the treatment of choice for gonorrheal cervicitis and is also given as a one time dose. A course of doxycycline (C) is an alternative therapy for chlamydial cervicitis, but azithromycin is preferred because of improved adherence of single-dose therapy.

A 24-year-old woman with no past medical history presents with left wrist pain after a fall. The left extremity is grossly deformed and the patient complains of severe pain. The patient has a blood pressure of 183/100 mm Hg. While awaiting X-rays, what management is indicated for the patients elevated blood pressure? Arrange admission for blood pressure control Start an oral beta-blocker and monitor for response Start intravenous beta-blocker and admit to the intensive care unit Treat the patient's pain and reassess the blood pressure

Correct Answer ( D ) Explanation: The patient presents with a markedly elevated blood pressure in the setting of pain from a trauma and should have pain control initiated and her blood pressure rechecked. Hypertension is defined as a persistent SBP >140 mm Hg or DBP >90 mm Hg. Pain and anxiety are common causes of elevated blood pressure and heart rate in the outpatient setting. Historically, patients with elevated blood pressure and nonspecific symptoms were referred to as hypertensive urgency but this term has fallen out of use. In a patient presenting with elevated blood pressure who does not have signs or symptoms of end-organ damage, the clinician's focus should be on identifying external reasons for the elevated pressure and treating or addressing these. In this case, the reduction or relief of pain will likely lead to decreased blood pressure. Patients with elevated blood pressure and an absence of end-organ damage (e.g. acute coronary syndrome, aortic dissection, encephalopathy, change in renal function) do not require admission (A) for management. A primary care physician in the outpatient setting best manages these patients. Starting a beta-blocker (B) will not be beneficial in a patient with acute pain as the cause of elevated blood pressure. Similarly, administration of an intravenous beta-blocker and admission to the intensive care unit (C) is not indicated as the patient exhibits no end-organ damage.

An 18-year-old man presents to the ED near where he resides in southern Missouri, complaining of fever and shortness of breath for two weeks. His temperature is 37.5°C, heart rate 107, blood pressure 115/76 mm Hg, respiratory rate 22 breaths per minute, and his oxygen saturation on room air is 94%. He is a construction worker whose hobbies include hiking, caving, and hunting. A chest radiograph shows multiple granulomas and hilar adenopathy. Which of the following organisms is most likely responsible for his presentation? Blastomyces dermatitides Candida glabrata Coccidioides immitis Histoplasma capsulatum Taenia saginata

Correct Answer ( D ) Explanation: This man has multiple factors in his history that are consistent with a fungal pulmonary infection caused by histoplasmosis. He lives in southern Missouri, the correct region; he also works in construction and has a hobby of caving. Histoplasmosis is typically linked to contact with bird or bat droppings or exposure to construction sites. It is found almost exclusively in the Mississippi and Ohio River valleys. The disseminated form of this illness can be asymptomatic or can cause a pneumonia-like illness. Findings on chest radiograph can be variable but often include hilar adenopathy. Blastomycosis (A) is found in Midwestern states bordering the Great Lakes. Candida glabrata (B) is a fungal infection also associated with a pneumonia-like illness, but it typically occurs only in immunocompromised patients. Coccidioidomycosis (C) is seen in the United States primarily in the Southwest and desert regions, especially California's San Joaquin Valley. Taenia saginata (E) is a type of tapeworm found in raw beef and is not associated with pulmonary complaints.

A 45-year-old man with a history of alcohol abuse presents with numbness and weakness of the left hand. He states he slept on a bench last night and awoke this morning with the symptoms. Physical examination reveals decreased sensation over the first, second, and third digits and a wrist drop. What management is indicated? CT of the cervical spine MRI of the brain Noncontrast CT scan of the head Wrist splint and follow up with neurology

Correct Answer ( D ) Explanation: This patient presents with a radial nerve palsy likely caused by direct compression and should have a wrist splint placed and given follow up with a neurologist. The radial nerve arises from the C5-T1 roots, exits the brachial plexus and passes into the spiral groove behind the proximal humerus. It bifurcates at the antecubital fossa. This nerve controls extension of the fingers, thumb and wrist. Compression of the radial nerve in the axilla occurs secondary to the improper use of crutches or from an extended period of unconsciousness with prolonged compression. This pathology is typically called a "Saturday night palsy." Patients with radial nerve palsy present with weakness in extension of the fingers and wrist. Spontaneous recovery often occurs but may take 2-4 months. During this time, the wrist should be splinted with 60 degrees of dorsiflexion in order to prevent atrophy and contractures. CT of the cervical spine (A), MRI of the brain (B) and CT scan of the head (C) are not indicated as this is a peripheral nerve injury.

A 35-year-old man presents to the emergency department at 3:00 am complaining of intense bilateral eye pain, redness, and tearing that woke him from sleep fifteen minutes prior to arrival. Eye examination with fluorescein staining of the corneas reveals diffuse punctate corneal lesions as shown above. Further history would likely reveal which of the following? Allergies to dust mites, tree pollen, pet dander, and cockroaches Continuous contact lens usage for the past four months Corrosive chemical exposure Welding without eye protection earlier in the day

Correct Answer ( D ) Explanation: Ultraviolet keratitis is the result of corneal epithelial damage from exposure to intense UV light. It can be from welding without eye protection, prolonged exposure to sunlight, or using a tanning bed without eye protection. Signs and symptoms include intense eye pain, photophobia, redness, and tearing with diffuse punctate corneal lesions seen with fluorescein staining. Onset of symptoms is typically delayed for several hours. Treatment consists of topical cycloplegics, broad spectrum antibiotic eye ointment, and ophthalmologic follow-up. Allergies to dust mites, tree pollen, pet dander, and cockroaches (A) is a feature of allergic conjunctivitis. Allergic conjunctivitis is characterized by bilateral eye itching and conjunctival injection with a normal fluorescein examination. Continuous contact lens usage for the past four months (B) is a cause of corneal ulcers. Corneal ulcers are epithelial defects typically caused by Pseudomonas aeruginosa in the setting of continuous contact lens usage. They present with pain, redness, photophobia, and tearing, but are usually unilateral and consist of a single corneal lesion. Corrosive chemical exposure (C) can potentially cause diffuse corneal damage, but would not present with a delay in onset of symptoms.


संबंधित स्टडी सेट्स

Cardio Pharm Qs - CHF, Diuretics, Anti-anginal, Anti-arrhythmics, Lipid Lowering

View Set

Ch. 12 HW Quiz: Social Psychology

View Set

Chapter 2: Bits, Data Types, and Operations

View Set

Sesión 1: Conceptos de Salud Pública y Medicina Preventiva

View Set

Quiz 4 & 5 Market Failures and the Role of the Government

View Set